LSAT and Law School Admissions Forum

Get expert LSAT preparation and law school admissions advice from PowerScore Test Preparation.

 Administrator
PowerScore Staff
  • PowerScore Staff
  • Posts: 8917
  • Joined: Feb 02, 2011
|
#91391
Complete Question Explanation

The correct answer choice is (D).

Answer choice (A):

Answer choice (B):

Answer choice (C):

Answer choice (D): This is the correct answer choice.

Answer choice (E):

This explanation is still in progress. Please post any questions below!
User avatar
 Fightforthat170
  • Posts: 16
  • Joined: Oct 22, 2021
|
#91786
Hi yall,

I was struggling between A and D, and eventually went with A. I thought they could both be true.

I eventually went with A because I thought the distinction made in D "algorithms and the specific way in which a software expresses them" is never made by "the proponents of the software patents" but only by the author. However, maybe I mis-interpreted something or misread some parts of the passage.

I actually have the same doubt for A. I don't think the patent proponents ever compared which part of the algorithms is more "inventive," however if we were to eliminate A on this premise, shouldn't we also eliminate D as well?

Please help!
User avatar
 atierney
PowerScore Staff
  • PowerScore Staff
  • Posts: 215
  • Joined: Jul 06, 2021
|
#91814
Hello,

Keep fighting man. We, at PowerScore, are definitely here for you! I agree you that A's comparison of the inventiveness between the computer code v. underlying algorithm not only is not supported in the passage, but also kind of runs counter to what one would to be the programmer's argument. Shouldn't the design of the process of the code itself, that is the underlying algorithm, be the more inventive aspect, since this would give it the stronger case for patent protection? It seems like A is somewhat of an opposite answer, and at the very lest, a Shell Answer.

Now, in terms of D, this would an answer that one might expect without really reading the passage. In other words, if I'm a programmer, I want as much protection as you can give me for my programs! Protect everything! Thus, you would have to have evidence that ran contrary to this somewhat sensible notion explicitly stated in the passage itself, to not select this answer.

And I think on those grounds, it's the easiest to see why D is the clear winner here. Let me know if you have further questions on this.
User avatar
 ashpine17
  • Posts: 321
  • Joined: Apr 06, 2021
|
#97904
isn't c iterally stated word for word in the passage though?
 Robert Carroll
PowerScore Staff
  • PowerScore Staff
  • Posts: 1787
  • Joined: Dec 06, 2013
|
#98276
ashpine17,

Where exactly is that stated word for word? I don't see it myself. Let us know!

Robert Carroll

Get the most out of your LSAT Prep Plus subscription.

Analyze and track your performance with our Testing and Analytics Package.